next up previous
Next: Applications Up: Apêndice: Two theorems by Previous: Introduction

The First Theorem

Actually, the Helmholtz theorem8proves a slightly different thing, from which the statement above follows immediately: a vector field $ \vec{V}$ which vanishes at the boundaries can be written as the sum of two terms, one of which is irrotational and the other, solenoidal (that is, divergenceless)[12]. Consider the following well-known identity for an arbitrary vector field $ \vec{Z}(\vec{r})$ :

$\displaystyle -\vec{\nabla}^2\vec{Z} = - \vec{\nabla}(\vec{\nabla}.\vec{Z}) + \vec{\nabla}\times\vec{\nabla}\times\vec{Z}$ (199)

If we now take our vector field to be

$\displaystyle \vec{V}=-\vec{\nabla}^2\vec{Z}$ (200)

then it follows that

$\displaystyle \vec{V}=-\vec{\nabla}U + \vec{\nabla}\times \vec{W}$ (201)

with

$\displaystyle U=\vec{\nabla}.\vec{Z}$ (202)

and

$\displaystyle \vec{W}=\vec{\nabla}\times \vec{Z}$ (203)

Eq.(201) is Helmholtz's theorem, as $ \vec{\nabla}U$ is irrotational and $ \vec{\nabla}\times \vec{W}$ is solenoidal.

But, is it general? It assumes that our vector field can be written as the Laplacian of some other one...
This constitutes, however, no problem if $ \vec{V}$ vanishes at infinity fast enough, for, then, the equation

$\displaystyle \vec{\nabla}^2\vec{Z}=-\vec{V} \;,$ (204)

which is Poisson's equation, has always the solution

$\displaystyle \vec{Z}(\vec{r})= \frac{1}{4\pi}\int d^3\vec{r'}\frac{\vec{V}(\vec{r'})} {\vert\vec{r}-\vec{r'}\vert}\;.$ (205)

It is now a simple matter to prove, from Eq.(201), that $ \vec{V}$ is determined from its $ div$ and $ curl$. Taking, in fact, the divergence of Eq.(201), we have:

$\displaystyle div \vec{V} = -\vec{\nabla}^2 U$ (206)

which is, again, Poisson's equation, and, so, determines $ U$ as

$\displaystyle U(\vec{r}) = \frac{1}{4\pi}\int d^3\vec{r'}\frac{\vec{\nabla}'.\vec{V}(\vec{r'})} {\vert\vec{r}-\vec{r'}\vert}$ (207)

Take now the $ curl$ of Eq.(201). We have
$\displaystyle \vec{\nabla}\times \vec{V}$ $\displaystyle =$ $\displaystyle \vec{\nabla}\times\vec{\nabla}\times\vec{W}$  
  $\displaystyle =$ $\displaystyle \vec{\nabla}(\vec{\nabla}.\vec{W}) - \vec{\nabla}^2\vec{W}$ (208)

Now, $ \vec{\nabla}.\vec{W}=0$, as $ \vec{W}=\vec{\nabla}\times\vec{Z}$, so another Poisson equation determines $ \vec{W}$. Using $ U$ and $ \vec{W}$ so determined in Eq.(201) proves our contention.
next up previous
Next: Applications Up: Apêndice: Two theorems by Previous: Introduction
Henrique Fleming 2003-08-11